Emergency Nursing Questions

Share Embed Donate


Short Description

se...

Description

Emergency Nursing Quiz - I This page was last updated on April 5, 2011

1. Which of the follwoing is the most effective benzodiazepine for treating status epilepticus and is the treatment of choice for controlling seizures acutely? A. Midazolam B. Oxazepam C. Flurazepam D. Lorazepam Answer Key 2. Subdiaphragmatic abdominal thrust performed to clear airway obstruction in the awake patient without ventilation is termed as: A. Leopold's maneuvers B. Pallach's maneuver C. Heimlich maneuver D. Valsalva maneuver Answer Key 3. A slight bluish discoloration around the navel, is a sign of hemoperitoneum, A. Cullen's sign B. Kernig’s sign C. Brudzinski’s signs D. Kehr's sign Answer Key 4. Signs of meningeal irritation include all the following, EXCEPT: A. Neck stiffness B. Brudzinski's sign C. Kernig’s sign D. Kehr's sign Answer Key 5. The APACHE II system is a: A. Quality assurance system

B. Severity-of-illness scoring system C. Computerized vital signsmonitoring system D. Neborn hydration monitoring system Answer Key 6. A client is admitted to the emergency department following an automobile accident. The client has four fractured ribs and a right sided pneumothorax. Which of the following respiratory assessment findings would the nurse expect to find? A. Crackles on the right chest and a respiratory rate of 8 breaths/minute. B. Diminished breath sounds on the right and pain on inspiration. C. Bilateral rhonchi and pink frothy sputum. D. Dry cough and wheezing on the right side of the chest. Answer Key 7. Clinical evidence of diminished Cardiac Output includes: A. a narrow pulse pressure B. rapid capillary refill C. widened pulse pressure D. bounding pulses Answer Key 8. The most common cause of high cardiac output hypotension is: A. hemorrhage B. cardiac failure C. sepsis D. neurogenic shock Answer Key 9. Intermediate Syndrome is observed in: A. cardiac arrest B. respiratory failure C. renal failure D. organophosphorous poisoning Answer Key

10. The procedure of compressing cricothyroid cartilage posteriorly against vertebral bodies during endotracheal intubation is called: A. Valsalva maneuver B. Sellick maneuver C. Heimlich maneuver D. Leopold's maneuvers Answer Key 11. Which of the following pairing about Glasgow Coma Score is wrong? A. 6 —Spontaneous Eye Opening B. 1—No Verbal Response C. 5—Localizes to pain (Motor Activity) D. 3—Eye Opening to Verbal stimuli Answer Key Answer Key

1.D

2. C

3. A

4. D

5. B

6. A

7. A

8. C

9. D

10. B

11. A

BACK TO TOP Back to Quiz Corner

1.

The nurse is triaging four clients injured in a train derailment. Which client should receive priority treatment? A. A 42-year-old with dyspnea and chest asymmetry B. A 17-year-old with a fractured arm C. A 4-year-old with facial lacerations D. A 30-year-old with blunt abdominal trauma

2.

Direct pressure to a deep laceration on the client’s lower leg has failed to stop the bleeding. The nurse’s next action should be to: A. Place a tourniquet proximal to the laceration. B. Elevate the leg above the level of the heart. C. Cover the laceration and apply an ice compress.

D. Apply pressure to the femoral artery. 3.

A pediatric client is admitted after ingesting a bottle of vitamins with iron. Emergency care would include treatment with: A. Acetylcysteine B. Deferoxamine C. Calcium disodium acetate D. British antilewisite

4.

The nurse is preparing to administer Ringer’s Lactate to a client with hypovolemic shock. Which intervention is important in helping to stabilize the client’s condition? A. Warming the intravenous fluids B. Determining whether the client can take oral fluids C. Checking for the strength of pedal pulses D. Obtaining the specific gravity of the urine

5.

The emergency room staff is practicing for its annual disaster drill. According to disaster triage, which of the following four clients would be cared for last? A. A client with a pneumothorax B. A client with 70% TBSA full thickness burns C. A client with fractures of the tibia and fibula D. A client with smoke inhalation injuries

6.

An unresponsive client is admitted to the emergency room with a history of diabetes mellitus. The client’s skin is cold and clammy, and the blood pressure reading is 82/56. The first step in emergency treatment of the client’s symptoms would be: A. Checking the client’s blood sugar B. Administering intravenous dextrose C. Intubation and ventilator support D. Administering regular insulin

7.

A client with a history of severe depression has been brought to the emergency room with an overdose of barbiturates. The nurse should pay careful attention to the client’s: A. Urinary output B. Respirations C. Temperature D. Verbal responsiveness

8.

A client is to receive antivenin following a snake bite. Before administering the antivenin, the nurse should give priority to: A. Administering a local anesthetic B. Checking for an allergic response C. Administering an anxiolytic D. Withholding fluids for 6–8 hours

9.

The nurse is caring for a client following a radiation accident. The client is determined to have incorporation. The nurse knows that the client will: A. Not need any medical treatment for radiation exposure B. Have damage to the bones, kidneys, liver, and thyroid C. Experience only erythema and desquamation D. Not be radioactive because the radiation passes through the body

10.

The emergency staff has undergone intensive training in the care of clients with suspected anthrax. The staff understands that the suggested drug for treating anthrax is: A. Ancef (cefazolin sodium) B. Cipro (ciprofloxacin) C. Kantrex (kanamycin) D. Garamycin (gentamicin)

Answer Rationales 1.

Answer A is correct. Following the ABCDs of basic emergency care, the client with dyspnea and asymmetrical chest should be cared for first because these symptoms are associated with flail chest. Answer D is incorrect because he should be cared for second because of the likelihood of organ damage and bleeding. Answer B is incorrect because he should be cared for after the client with abdominal trauma. Answer C is incorrect because he should receive care last because his injuries are less severe.

2.

Answer B is correct. If bleeding does not subside with direct pressure, the nurse should elevate the extremity above the level of the heart. Answers A and D are done only if other measures are ineffective, so they are incorrect. Answer C would slow the bleeding but will not stop it, so it’s incorrect.

3.

Answer B is correct. Deferoxamine is the antidote for iron poisoning. Answer A is the antidote for acetaminophen overdose, making it wrong. Answers C and D are antidotes for lead poisoning, so they are wrong.

4.

Answer A is correct. Warming the intravenous fluid helps to prevent further stress on the vascular system. Thirst is a sign of hypovolemia; however, oral fluids alone

will not meet the fluid needs of the client in hypovolemic shock, so answer B is incorrect. Answers C and D are wrong because they can be used for baseline information but will not help stabilize the client. 5.

Answer B is correct. The client with 70% TBSA burns would be classified as an emergent client. In disaster triage, emergent clients, code black, are cared for last because they require the greatest expenditure of resources. Answers A and D are examples of immediate clients and are assigned as code red, so they are wrong. These clients are cared for first because they can survive with limited interventions. Answer C is wrong because it is an example of a delayed client, code yellow. These clients have significant injuries that require medical care.

6.

Answer A is correct. The client has symptoms of insulin shock and the first step is to check the client’s blood sugar. If indicated, the client should be treated with intravenous dextrose. Answer B is wrong because it is not the first step the nurse should take. Answer C is wrong because it does not apply to the client’s symptoms. Answer D is wrong because it would be used for diabetic ketoacidosis, not insulin shock.

7.

Answer B is correct. Barbiturate overdose results in central nervous system depression, which leads to respiratory failure. Answers A and C are important to the client’s overall condition but are not specific to the question, so they are incorrect. The use of barbiturates results in slow, slurred speech, so answer D is expected, and therefore incorrect.

8.

Answer B is correct. The nurse should perform the skin or eye test before administering antivenin. Answers A and D are unnecessary and therefore incorrect. Answer C would help calm the client but is not a priority before giving the antivenin, making it incorrect.

9.

Answer B is correct. The client with incorporation radiation injuries requires immediate medical treatment. Most of the damage occurs to the bones, kidneys, liver, and thyroid. Answers A, C, and D refer to external irradiation, so they are wrong.

10.

Answer B is correct. Cipro (ciprofloxacin) is the drug of choice for treating anthrax. Answers A, C, and D are not used to treat anthrax, so they are incorrect.

1. You are the charge nurse in an emergency department (ED) and must assign two staff members to cover the triage area. Which team is the most appropriate for this assignment? a. An advanced practice nurse and an experienced LPN/LVN b. An experienced LPN/LVN and an inexperienced RN c. An experienced RN and an inexperienced RN d. An experienced RN and a nursing assistant

2. You are working in the triage area of an ED, and four patients approach the triage desk at the same time. List the order in which you will assess these patients. a. An ambulatory, dazed 25-year-old male with a bandaged head wound b. An irritable infant with a fever, petechiae, and nuchal rigidity c. A 35-year-old jogger with a twisted ankle, having pedal pulse and no deformity d. A 50-year-old female with moderate abdominal pain and occasional vomiting 3. In conducting a primary survey on a trauma patient, which of the following is considered one of the priority elements of the primary survey? a. Complete set of vital signs b. Palpation and auscultation of the abdomen c. Brief neurologic assessment d. Initiation of pulse oximetry 4. A 56-year-old patient presents in triage with left-sided chest pain, diaphoresis, and dizziness. This patient should be prioritized into which category? a. High urgent b. Urgent c. Non-urgent d. Emergent 5. The physician has ordered cooling measures for a child with fever who is likely to be discharged when the temperature comes down. Which of the following would be appropriate to delegate to the nursing assistant? a. Assist the child to remove outer clothing. b. Advise the parent to use acetaminophen instead of aspirin. c. Explain the need for cool fluids. d. Prepare and administer a tepid bath. 6. It is the summer season, and patients with signs and symptoms of heat-related illness present in the ED. Which patient needs attention first?

a. An elderly person complains of dizziness and syncope after standing in the sun for several hours to view a parade b. A marathon runner complains of severe leg cramps and nausea. Tachycardia, diaphoresis, pallor, and weakness are observed. c. A previously healthy homemaker reports broken air conditioner for days. Tachypnea, hypotension, fatigue, and profuse diaphoresis are observed. d. A homeless person, poor historian, presents with altered mental status, poor muscle coordination, and hot, dry, ashen skin. Duration of exposure is unknown. 7. You respond to a call for help from the ED waiting room. There is an elderly patient lying on the floor. List the order for the actions that you must perform. a. Perform the chin lift or jaw thrust maneuver. b. Establish unresponsiveness. c. Initiate cardiopulmonary resuscitation (CPR). d. Call for help and activate the code team. e. Instruct a nursing assistant to get the crash cart. 8. The emergency medical service (EMS) has transported a patient with severe chest pain. As the patient is being transferred to the emergency stretcher, you note unresponsiveness, cessation of breathing, and no palpable pulse. Which task is appropriate to delegate to the nursing assistant? a. Chest compressions b. Bag-valve mask ventilation c. Assisting with oral intubation d. Placing the defibrillator pads 9. An anxious 24-year-old college student complains of tingling sensations, palpitations, and chest tightness. Deep, rapid breathing and carpal spasms are noted. What priority nursing action should you take? a. Notify the physician immediately. b. Administer supplemental oxygen. c. Have the student breathe into a paper bag. d. Obtain an order for an anxiolytic medication.

10.An experienced traveling nurse has been assigned to work in the ED; however, this is the nurse’s first week on the job. Which area of the ED is the most appropriate assignment for the nurse? a. Trauma team b. Triage c. Ambulatory or fats track clinic d. Pediatric medicine team 11.A tearful parent brings a child to the ED for taking an unknown amount of children’s chewable vitamins at an unknown time. The child is currently alert and asymptomatic. What information should be immediately reported to the physician? a. The ingested children’s chewable vitamins contain iron. b. The child has been treated several times for ingestion of toxic substances. c. The child has been treated several times for accidental injuries. d. The child was nauseated and vomited once at home. 12.In caring for a victim of sexual assault, which task is most appropriate for an LPN/LVN? a. Assess immediate emotional state and physical injuries b. Collect hair samples, saliva swabs, and scrapings beneath fingernails. c. Provide emotional support and supportive communication. d. Ensure that the “chain of custody” is maintained. 13.You are caring for a victim of frostbite to the feet. Place the following interventions in the correct order. a. Apply a loose, sterile, bulky dressing. b. Give pain medication. c. Remove the victim from the cold environment. d. Immerse the feet in warm water 100o F to 105o F (40.6o C to 46.1o C) 14.A patient sustains an amputation of the first and second digits in a chainsaw accident. Which task should be delegated to the LPN/LVN? a. Gently cleanse the amputated digits with Betadine solution.

b. Place the amputated digits directly into ice slurry. c. Wrap the amputated digits in sterile gauze moistened with saline. d. Store the amputated digits in a solution of sterile normal saline. 15.A 36-year-old patient with a history of seizures and medication compliance of phenytoin (Dilantin) and carbamazepine (Tegretol) is brought to the ED by the MS personnel for repetitive seizure activity that started 45 minutes prior to arrival. You anticipate that the physician will order which drug for status epilepticus? a. PO phenytoin and carbamazepine b. IV lorazepam (Ativan) c. IV carbamazepam d. IV magnesium sulfate 16.You are preparing a child for IV conscious sedation prior to repair of a facial laceration. What information should you immediately report to the physician? a. The parent is unsure about the child’s tetanus immunization status. b. The child is upset and pulls out the IV. c. The parent declines the IV conscious sedation. d. The parent wants information about the IV conscious sedation. 17.An intoxicated patient presents with slurred speech, mild confusion, and uncooperative behavior. The patient is a poor historian but admits to “drinking a few on the weekend.” What is the priority nursing action for this patient? a. Obtain an order for a blood alcohol level. b. Contact the family to obtain additional history and baseline information. c. Administer naloxone (Narcan) 2 – 4 mg as ordered. d. Administer IV fluid support with supplemental thiamine as ordered. 18.When an unexpected death occurs in the ED, which of the following tasks is most appropriate to delegate to the nursing assistant? a. Escort the family to a place of privacy. b. Go with the organ donor specialist to talk to the family.

c. Assist with postmortem care. d. Assist the family to collect belongings. 19.Following emergency endotracheal intubation, you must verify tube placement and secure the tube. List in order the steps that are required to perform this function? a. Obtain an order for a chest x-ray to document tube placement. b. Secure the tube in place. c. Auscultate the chest during assisted ventilation. d. Confirm that the breath sounds are equal and bilateral. _____, _____, _____, _____ 20.A teenager arrives by private car. He is alert and ambulatory, but this shirt and pants are covered with blood. He and his hysterical friends are yelling and trying to explain that that they were goofing around and he got poked in the abdomen with a stick. Which of the following comments should be given first consideration? a. “There was a lot of blood and we used three bandages.” b. “He pulled the stick out, just now, because it was hurting him.” c. “The stick was really dirty and covered with mud.” d. “He’s a diabetic, so he needs attention right away.” 21.A prisoner, with a known history of alcohol abuse, has been in police custody for 48 hours. Initially, anxiety, sweating, and tremors were noted. Now, disorientation, hallucination, and hyper-reactivity are observed. The medical diagnosis is delirium tremens. What is the priority nursing diagnosis? a. Risk for Injury related to seizures b. Risk for Other-Directed Violence related to hallucinations c. Risk for Situational Low Self-esteem related to police custody d. Risk for Nutritional Deficit related to chronic alcohol abuse 22.You are assigned to telephone triage. A patient who was stung by a common honey bee calls for advice, reports pain and localized swelling, but denies any respiratory distress or other systemic signs of anaphylaxis. What is the action that you should direct the caller to perform? a. Call 911. b. Remove the stinger by scraping. c. Apply a cool compress.

d. Take an oral antihistamine. 23.In relation to submersion injuries, which task is most appropriate to delegate to an LPN/LVN? a. Talk to a community group about water safety issues. b. Stabilize the cervical spine for an unconscious drowning victim. c. Remove wet clothing and cover the victim with a warm blanket. d. Monitor an asymptomatic near-drowning victim. 24.You are assessing a patient who has sustained a cat bite to the left hand. The cat is up-to-date immunizations. The date of the patient’s last tetanus shot is unknown. Which of the following is the priority nursing diagnosis? a. Risk for Infection related to organisms specific to cat bites b. Impaired Skin Integrity related to puncture wounds c. Ineffective Health Maintenance related to immunization status d. Risk for Impaired Mobility related to potential tendon damage 25.These patients present to the ED complaining of acute abdominal pain. Prioritize them in order of severity. a. A 35-year-old male complaining of severe, intermittent cramps with three episodes of watery diarrhea, 2 hours after eating b. A 11-year-old boy with a low-grade fever, left lower quadrant tenderness, nausea, and anorexia for the past 2 days c. A 40-year-old female with moderate left upper quadrant pain, vomiting small amounts of yellow bile, and worsening symptoms over the past week d. A 56-year-old male with a pulsating abdominal mass and sudden onset of pressure-like pain in the abdomen and flank within the past hour _____, _____, _____, _____ 26.The nursing manager decides to form a committee to address the issue of violence against ED personnel. Which combination of employees is best suited to fulfill this assignment? a. ED physicians and charge nurses b. Experienced RNs and experienced paramedics c. RNs, LPN/LVNs, and nursing assistants d. At least one representative from each group of ED personnel

27.In a multiple-trauma victim, which assessment finding signals the most serious and life-threatening condition? a. A deviated trachea b. Gross deformity in a lower extremity c. Decreased bowel sounds d. Hematuria 28.A patient in a one-car rollover presents with multiple injuries. Prioritize the interventions that must be initiated for this patient. a. Secure/start two large-bore IVs with normal saline b. Use the chin lift or jaw thrust method to open the airway. c. Assess for spontaneous respirations d. Give supplemental oxygen per mask. e. Obtain a full set of vital signs. f. Remove patient’s clothing. g. Insert a Foley catheter if not contraindicated. _____, _____, _____, _____, ____, ____, ____ 29.In the work setting, what is your primary responsibility in preparing for disaster management that includes natural disasters or bioterrorism incidents? a. Knowledge of the agency’s emergency response plan b. Awareness of the signs and symptoms for potential agnets of bioterrorism c. Knowledge of how and what to report to the CDC d. Ethical decision-making about exposing self to potentially lethal substances 30.You are giving discharge instructions to a woman who has been treated for contusions and bruises sustained during an episode of domestic violence. What is your priority intervention for this patient? a. Transportation arrangements to a safe house b. Referral to a counselor c. Advise about contacting the police

d. Follow-up appointment for injuries

-----------------------------------------------------------------------------------1. Answer: C – Triage requires at least one experienced RN. Pairing an experienced RN with inexperienced RN provides opportunities for mentoring. Advanced practice nurses are qualified to perform triage; however, their services are usually required in other areas of the ED. An LPN/LVN is not qualified to perform the initial patient assessment or decision making. Pairing an experienced RN with a nursing assistant is the second best option, because the assistant can obtain vital signs and assist in transporting. 2. Answer: B, A, D, C – An irritable infant with fever and petechiae should be further assessed for other meningeal signs. The patient with the head wound needs additional history and assessment for intracranial pressure. The patient with moderate abdominal pain is uncomfortable, but not unstable at this point. For the ankle injury, medical evaluation can be delayed 24 – 48 hours if necessary. 3. Answer: C – A brief neurologic assessment to determine level of consciousness and pupil reaction is part of the primary survey. Vital signs, assessment of the abdomen, and initiation of pulse oximetry are considered part of the secondary survey. 4. Answer: D – Chest pain is considered an emergent priority, which is defined as potentially lifethreatening. Patients with urgent priority need treatment within 2 hours of triage (e.g. kidney stones). Non-urgent conditions can wait for hours or even days. (High urgent is not commonly used; however, in 5-tier triage systems, High urgent patients fall between emergent and urgent in terms of the time lapsing prior to treatment). 5. Answer: A – The nursing assistant can assist with the removal of the outer clothing, which allows the heat to dissipate from the child’s skin. Advising and explaining are teaching functions that are the responsibility of the RN. Tepid baths are not usually performed because of potential for rebound and shivering. 6. Answer: D – The homeless person has symptoms of heat stroke, a medical emergency, which increases risk for brain damage. Elderly patients are at risk for heat syncope and should be educated to rest in cool area and avoid future similar situations. The runner is having heat crams, which can be managed with rest and fluids. The housewife is experiencing heat exhaustion, and management includes fluids (IV or parenteral) and cooling measures. The prognosis for recovery is good. 7. Answer: B, D, A, C, E – Establish unresponsiveness first. (The patient may have fallen and sustained a minor injury.) If the patient is unresponsive, get help and have someone initiate the code. Performing the chin lift or jaw thrust maneuver opens the airway. The nurse is then responsible for starting CPR. CPR should not be interrupted until the patient recovers or it is determined that heroic efforts have been exhausted. A crash cart should be at the site when the code team arrives; however, basic CPR can be effectively performed until the team arrives.

8. Answer: A – Nursing assistants are trained in basic cardiac life support and can perform chest compressions. The use of the bag-valve mask requires practice and usually a respiratory therapist will perform this function. The nurse or the respiratory therapist should provide PRN assistance during intubation. The defibrillator pads are clearly marked; however, placement should be done by the RN or physician because of the potential for skin damage and electrical arcing. 9. Answer: C – The patient is hyperventilating secondary to anxiety, and breathing into a paper bag will allow rebreathing of carbon dioxide. Also, encouraging slow breathing will help. Other treatments such as oxygen and medication may be needed if other causes are identified. 10. Answer: C – The fast track clinic will deal with relatively stable patients. Triage, trauma, and pediatric medicine should be staffed with experienced nurses who know the hospital routines and policies and can rapidly locate equipment. 11. Answer: A – Iron is a toxic substance that can lead to massive hemorrhage, coma, shock, and hepatic failure. Deferoxame is an antidote that can be used for severe cases of iron poisoning. Other information needs additional investigation, but will not change the immediate diagnostic testing or treatment plan. 12. Answer: C – The LPN/LVN is able to listen and provide emotional support for her patients. The other tasks are the responsibility of an RN or, if available, a SANE (sexual assault nurse examiner) who has received training to assess, collect and safeguard evidence, and care for these victims. 13. Answer: C, B, D, A – The victim should be removed from the cold environment first, and then the rewarming process can be initiated. It will be painful, so give pain medication prior to immersing the feet in warmed water. 14. Answer: C – The only correct intervention is C. the digits should be gently cleansed with normal saline, wrapped in sterile gauze moistened with saline, and placed in a plastic bag or container. The container is then placed on ice. 15. Answer: B – IV Lorazepam (Ativan) is the drug of choice for status epilepticus. Tegretol is used in the management of generalized tonic-clonic, absence or mixed type seizures, but it does not come in an IV form. PO (per os) medications are inappropriate for this emergency situation. Magnesium sulfate is given to control seizures in toxemia of pregnancy. Medical-Surgical Nursing 16. Answer: C – Parent refusal is an absolute contraindication; therefore, the physician must be notified. Tetanus status can be addressed later. The RN can restart the IV and provide information about conscious sedation; if the parent still notsatisfied, the physician can give more information. 17. Answer: D – The patient presents with symptoms of alcohol abuse and there is a risk for Wernicke’s syndrome, which is caused by a thiamine deficiency. Multiples drug abuse is not uncommon; however, there is nothing in the question that suggests an opiate overdose that requires naloxone. Additional information or the results of the blood alcohol level are part of the total treatment plan but should not delay the immediate treatment.

18. Answer: C – Postmortem care requires some turning, cleaning, lifting, etc., and the nursing assistant is able to assist with these duties. The RN should take responsibility for the other tasks to help the family begin the grieving process. In cases of questionable death, belongings may be retained for evidence, so the chain of custody would have to be maintained. 19. Answer: C, D, B, A – Auscultating and confirming equal bilateral breath sounds should be performed in rapid succession. If the sounds are not equal or if the sounds are heard over the mid-epigastric area, tube placement must be corrected immediately. Securing the tube is appropriate while waiting for the xray study. 20. Answer: B – An impaled object may be providing a tamponade effect, and removal can precipitate sudden hemodynamic decompensation. Additional history including a more definitive description of the blood loss, depth of penetration, and medical history should be obtained. Other information, such as the dirt on the stick or history of diabetes, is important in the overall treatment plan, but can be addressed later. 21. Answer: A – The patient demonstrates neurologic hyperactivity and is on the verge of a seizure. Patient safety is the priority. The patient needs chlordiazepoxide (Librium) to decrease neurologic irritability and phenytoin (Dilantin) for seizures. Thiamine and haloperidol (Haldol) will also be ordered to address the other problems. The other diagnoses are pertinent but not as immediate. 22. Answer: B – The stinger will continue to release venom into the skin, so prompt removal of the stinger is advised. Cool compresses and antihistamines can follow. The caller should be further advised about symptoms that require 911 assistance. 23. Answer: D – The asymptomatic patient is currently stable but should be observed for delayed pulmonary edema, cerebral edema, or pneumonia. Teaching and care of critical patients is an RN responsibility. Removing clothing can be delegated to a nursing assistant. 24. Answer: A – Cat’s mouths contain a virulent organism, Pasteurella multocida, that can lead to septic arthritis or bacteremia. There is also a risk for tendon damage due to deep puncture wounds. These wounds are usually not sutured. A tetanus shot can be given before discharge. 25. Answer: D, B, C, A – The patient with a pulsating mass has an abdominal aneurysm that may rupture and he may decompensate suddenly. The 11-year-old boy needs evaluation to rule out appendicitis. The woman needs evaluation for gallbladder problems that appear to be worsening. The 35-year-old man has food poisoning, which is usually self-limiting. 26. Answer: D – At least one representative from each group should be included because all employees are potential targets fro violence in the ED. 27. Answer: A – A deviated trachea is a symptoms of tension pneumothorax. All of the other symptoms need to be addressed, but are of lesser priority.

28. Answer: C, B, D, A, E, F, G – For a multiple trauma victim, many interventions will occur simultaneously as team members assist in the resuscitation. Methods to open the airway such as the chin lift or jaw thrust can be used simultaneously while assessing for spontaneous respirations. However, airway and oxygenation are priority. Starting IVs for fluid resuscitation is part of supporting circulation. (EMS will usually establish at least one IV in the field.) Nursing assistants can be directed to take vitals and remove clothing. Foley catheter is necessary to closely monitor output. 29. Answer: A – In preparing for disasters, the RN should be aware of the emergency response plan. The plan gives guidance that includes roles of team members, responsibilities, and mechanisms of reporting. Signs and symptoms of many agents will mimic common complaints, such as flu-like symptoms. Discussions with colleagues and supervisors may help the individual nurse to sort through ethical dilemmas related to potential danger to self. 30. Answer: A – Safety is a priority for this patient, and she should not return to a place where violence could reoccur. The other options are important for the long term management of this care. Read more at Nurseslabs.com Medical-Surgical Nursing Exam 15: Emergency Nursing (30 Items) http://nurseslabs.com/medical-surgical-nursing-exam-15-emergency-nursing-30-items/2/ 1. A patient arrives at the emergency department complaining of mid-sternal chest pain. Which of the following nursing action should take priority? A. A complete history with emphasis on preceding events. B. An electrocardiogram. C. Careful assessment of vital signs. D. Chest exam with auscultation.

1.

A nursing student is studying about disasters and emergency preparedness. Which of the following statements by the nursing student depicts a correct understanding of the difference between a disaster and an emergency? Your Answer: “An emergency is an unforeseen combination of circumstances calling for immediate action for a range of victims.” Rationale: # 1 is incorrect because disasters may be natural or man-made. # 3 is incorrect because man-made disasters are either accidental or intentional. # 4 is incorrect because natural disasters, not emergencies, are caused by acts of nature or emerging diseases. Nursing Process: Assessment Client Need: Safe, Effective Care Environment Cognitive Level: Application Objective: Distinguish the difference between an emergency and a disaster.

Strategy: Look at each statement to see if it clearly defines either a disaster or emergency. Then select the correct answer.

2.

An emergency room nurse is working when there is a bioterrorism attack in the city. Which of the following statements is a correct with regard to injuries or symptoms associated with a bioterrorism attack? Your Answer: It is not uncommon for the results of a biological attack to be made known several hours or days after the attack. Rationale: Biological terrorism is the use of etiological agents (disease) to cause harm or kill a population, food, and/or livestock. # 1 is incorrect because the main purpose of biological weapon use is mass devastation. # 3 is incorrect because a biological attack may not be known for several hours or days after the attack. # 4 is incorrect because detection is difficult as clients go to a number of different health care facilities for treatment. Nursing Process: Planning Client Need: Safe, Effective Care Environment Cognitive Level: Analysis Objective: Describe the types of injuries or symptoms that are associated with biological, chemical, or radiological terrorism. Strategy: Examine each statement to look for a correct statement about bioterrorism.

3.

The nurse is caring for a client with a blast injury. Which of the following nursing assessments would be most appropriate for this client? Your Answer: Assess for vasovagal hypotension Rationale: Blast injuries are the result of explosive munitions, often involving car or package bombs. Care for persons injured by blast injuries typically focuses on abdominal and lung injuries, penetrating wounds, traumatic amputations, and burns. # 2 is incorrect because a concussion, closed and open brain injury, stroke, spinal cord injury, and an air embolism-induced injury could result from a blast injury. #3 is incorrect because asphyxia could result from a hurricane injury. # 4 is incorrect because the client would have hypovolemia as a result from a blast injury. Nursing Process: Assessment Client Need: Safe, Effective Care Environment Cognitive Level: Analysis

Objective: Evaluate nursing interventions related to the treatment of injuries related to biological, chemical, or radiological terrorism. Strategy: Differentiate between the different types of common injuries and the associated assessments necessary to care for the client.

4.

An emergency room nurse is working when an Amtrak train derails. The emergency room nurse knows that reverse triage may need to be instituted. What is the rationale for using reverse triage? Your Answer: Reverse triage works on the principle of the greatest good for the greatest number. Rationale: During a disaster, nurses may be expected to perform triage. Triage means sorting. # 1 is incorrect because a mass casualty is an event with more than 100 victims. # 2 is incorrect because it describes basic triage and not reverse triage. # 3 is incorrect because victims least likely to survive or are already dead are color-coded as black. Nursing Process: Planning Client Need: Safe, Effective Care Environment Cognitive Level: Application Objective: Explain the rationale for reverse triage in disasters versus conventional triage in emergencies. Strategy: Read each answer choice to decide which statement correctly depicts the concepts of reverse and conventional triage.

5.

There has been a radioactive explosion nearby. The emergency room nurse must triage and manage the decontamination of the clients systematically. Which of the following clients would be decontaminated first? Your Answer: A client with the least injuries. Rationale: Reverse triage works on the principle of the greatest good for the greatest number. In this case, those persons who are the most ambulatory and least injured would be instructed to move quickly to the warm zone, away from the immediate accident site to get decontaminated and processed first. Those with minor injuries would be decontaminated next. Those with more severe to most severe injuries would be treated in that order. Nursing Process: Implementation Client Need: Safe, Effective Care Environment Cognitive Level: Application

Objective: Discuss situations requiring the need for client isolation or client decontamination. Strategy: Determine which client needs to be decontaminated first based on reverse triage principles.

6.

A preceptor is teaching a graduate nurse the concepts of mitigation. Which of the following statements, if made by the graduate nurse, would indicate an understanding of this concept? Your Answer: “A key nursing activity related to mitigation is the active participation in learning about the major aspects of disasters.” Rationale: Health care professionals are among the essential Nursing Process: Planning Client Need: Safe, Effective Care Environment Cognitive Level: Application Objective: Discuss the role of the nurse in disaster planning, response, and mitigation. Strategy: Understand and be able to define the concepts of disaster planning, response, and mitigation. Utilize these definitions in order to select the correct answer.

7.

A military nurse is working in Iraq. Because of the potential threat of hazardous gas, which of the following should be worn when working in a dangerous war zone? Select all that apply. Your Answer: A gas mask Protective clothing A hood, helmet, or headgear Rationale: Gas masks are used in a broad range of military, industrial, and emergency situations to protect the user from hazardous dust, gas, or other aerosols. Biological contaminants that are spread through aerosolized droplets create a threat to those not wearing personal protective equipment. A gas mask may be considered as a high-performance respirator, usually equipped with both eye protection and air supply protection or treatment. A hood, helmet, or headgear is generally worn to protect the skin, eyes, airways, and respiratory systems. Protective clothing is made to guard against mild irritants to serious lethal materials. Some protective suits are

disposable, intended for one use only. Others are durable, multi-layered fabrics, are completely impermeable and are reusable. Sunglasses and a surgical mask will not provide enough protection in this instance. Nursing Process: Planning Client Need: Safe, Effective Care Environment Cognitive Level: Application Objective: Discuss the role of the nurse in disaster planning, response, and mitigation. Strategy: Determine if each item would be indicated in this situation. Multiple answers are correct.

8.

An emergency room nurse is at work when a major terrorist attack occurs. In addition to caring for injured clients, the nurse must control the crowd. Which of the following statements, if made by the nurse, demonstrates an understanding of the concept of crowd control? Your Answer: “The agency’s security personnel and/or the local police force must control these crowds.” Rationale: When a disaster occurs, many people converge on the site. Those who come are the curious and those who truly mean to assist in the rescue and recovery of victims. However, this crowd of people needs to be controlled by authorities in charge of the site and rescue and recovery. #1 is incorrect because the job of crowd control is not under the auspices of the nurse. # 2 is incorrect because chaos ensues when the crowd is not maintained. # 4 is incorrect because nurses should not enter an area that has not been secured. Nursing Process: Planning Client Need: Safe, Effective Care Environment Cognitive Level: Application Objective: Discuss the role of the nurse in disaster planning, response, and mitigation. Strategy: Determine if the principles of crowd control are demonstrated in each answer choice. Eliminate each answer choice that incorrectly describes crowd control.

9.

A newly graduated nurse is learning about the nurse’s role in disaster relief as part of an orientation to the hospital. Which of the following concepts is accurate?

Your Answer: Nurses may have to assume expanded roles in making decisions for the most appropriate treatment of casualties. Rationale: Nurses must be aware of the roles nurses play in all aspects of disaster preparedness and response. #1 is incorrect because learning about disasters is essential. #2 is incorrect because the nurse’s role will be active, not passive. #3 is incorrect because basic skills should be applied. Nursing Process: Planning Client Need: Safe, Effective Care Environment Cognitive Level: Analysis Objective: Discuss the role of the nurse in disaster planning, response, and mitigation. Strategy: Examine the role of the nurse in disaster planning.

10.

A nursing student is learning about how to manage immunocompromised clients in a disaster situation. Which of the following statements made by the nursing student demonstrates an understanding of this concept? Your Answer: “A compromised immune system may be due to treatments such as chemotherapy, those who have had organ or bone marrow transplants, or from an underlying disease such as HIV.” Rationale: Clients who are immunocompromised pose special problems to the health care community especially if these persons are unable to access health care quickly in a disaster situation. # 1 is incorrect because the risk is greater. # 3 is incorrect because the vesicles or pustules will be distant from the vaccination site. # 4 is incorrect because bone marrow transplant clients need to avoid fresh fruits and vegetables. Nursing Process: Assessment Client Need: Safe, Effective Care Environment Cognitive Level: Analysis Objective: Identify ways that nurses are able to provide care to clients with special considerations. Strategy: Determine if each statement is correct with regard to caring for the immunocompromised client.

1. The nurse is teaching a class on biological warfare. Which information should the nurseinclude in the presentation?

a. Contaminated water is the only source of transmission of biological agents. b. Vaccines are available and being prepared to counteract biological agents. c. Biological weapons are less of a threat than chemical agents. d. Biological weapons are easily obtained and result in significant mortality. 2. Which signs/symptoms would the nurse assess in the client who has been exposed to theanthrax bacillus via the skin? a. A scabby, clear fluid±filled vesicle. b. Edema, pruritus, and a 2-mm ulcerated vesicle. c. Irregular brownish-pink spots around the hairline. d. Tiny purple spots flush with the surface of the skin. 3. The client has expired secondary to smallpox. Which information about funeral arrangementsis most important for the nurse to provide to the client¶s family? a. The client must be cremated. b. Suggest an open casket funeral. c. Bury the client within 24 hours. d. Notify the public health department. 4. A chemical exposure has just occurred at an airport. An off-duty nurse, knowledgeable aboutbiochemical agents, is giving directions to the travelers. Which direction should the nurseprovide to the travelers? a. Hold their breath as much as possible. b. Stand up to avoid heavy exposure. c. Lie down to stay under the exposure. d. Attempt to breathe through their clothing. 5. The nurse is caring for a client in the prodromal phase of radiation exposure. Whichsigns/symptoms would the nurse assess in the client? a. Anemia, leukopenia, and thrombocytopenia. b. Sudden fever, chills, and enlarged lymph nodes.

c. Nausea, vomiting, and diarrhea. d. Flaccid paralysis, diplopia, and dysphagia. 6. The off-duty nurse hears on the television of a bioterrorism act in the community.Which action should the nurse take first? a. Immediately report to the hospital emergency room. b. Call the American Red Cross to find out where to go. c. Pack a bag and prepare to stay at the hospital. d. Follow the nurse¶s hospital policy for responding. 7. Which situation would warrant the nurse obtaining information from a material safety datasheet (MSDS)? a. The custodian spilled a chemical solvent in the hallway. b. A visitor slipped and fell on the floor that had just been mopped. c. A bottle of antineoplastic agent broke on the client¶s floor. d. The nurse was stuck with a contaminated needle in the client¶s room. 8. The triage nurse is working in the emergency department. Which client should be assessedfirst? a. The 10-year-old child whose dad thinks the child¶s leg is broken. b. The 45-year-old male who is diaphoretic and clutching his chest. c. The 58-year-old female complaining of a headache and seeing spots. d. The 25-year-old male who cut his hand with a hunting knife. 9. According to the North Atlantic Treaty Organization (NATO) triage system, which situationwould be considered a level red (Priority 1)? a. Injuries are extensive and chances of survival are unlikely. b. Injuries are minor and treatment can be delayed hours to days. c. Injuries are significant but can wait hours without threat to life or limb. d. Injuries are life threatening but survivable with minimal interventions. 10. Which statement best describes the role of the medical-surgical nurse during a disaster? a. The nurse may be assigned to ride in the ambulance.

b. The nurse may be assigned as a first assistant in the operating room. c. The nurse may be assigned to crowd control. d. The nurse may be assigned to the emergency department.Nursing

Board Exam Review Questions in Emergency Part 6/20(ANSWER KEY) 1. Answer: DRationale: Because of the variety of agents, the means of transmission, and lethality of theagents, biological weapons, including anthrax, smallpox, and plague, is especially dangerous. 2. Answer: BRationale: Exposure to anthrax bacilli via the skin results in skin lesions, which cause edemawith pruritus and the formation of macules or papules that ulcerate, forming a 1-3 mm vesicle.Then a painless eschar develops, which falls off in one (1) to 2 weeks. 3. Answer: ARationale: Cremation is recommended because the virus can stay alive in the scabs of the bodyfor 13 years. 4. Answer: BRationale: Standing up will avoid heavy exposure the chemical will sink toward the floor orground. 5. Answer: CRationale: The prodromal phase (presenting symptoms) of radiation exposure occurs 48±72hours after exposure and the signs/symptoms are nausea, vomiting, diarrhea, anorexia, andfatigue. Higher exposures of radiation signs/symptoms include fever, respiratory distress, andexcitability. 6. Answer: DRationale: The nurse should follow the hospital¶s policy. Many times nurses will stay at homeuntil decisions are made as to where the employees should report. 7. Answer: ARationale: The MSDS provides chemical information regarding specific agents, healthinformation, and spill information for a variety of chemicals. It is required for every chemicalthat is found in the hospital. 8. Answer: BRationale: The triage nurse should see this client first because these are symptoms of a myocar-dial infarction, which potentially life is threatening. 9. Answer: DRationale: This is called the immediate category. Individuals in this group can progress rapidlyto expectant if treatment is delayed. 10. Answer: DRationale: New settings and atypical roles for nurses may be required during disasters; medical-surgical nurses can provide first aid and be required to work in unfamiliar settings.

Nursing Board Exam Review Questions in Emergency Part 5/20 1. Which intervention is the most important for the nurse to implement when performing mouth-tomouth resuscitation on a client who has pulseless ventricular fibrillation? a. Perform the jaw thrust maneuver to open the airway. b. Use the mouth to cover the client¶s mouth and nose. c. Insert an oral airway prior to performing mouth to mouth. d. Use a pocket mouth shield to cover client¶s mouth. 2. The nurse is teaching CPR to a class. Which statement best explains the definition of suddencardiac death? a. Cardiac death occurs after being removed from a mechanical ventilator. b. Cardiac death is the time that the physician officially declares the client dead. c. Cardiac death occurs within one (1) hour of the onset of cardiovascular symptoms. d. The death is caused by myocardial ischemia resulting from coronary artery disease. 3. Which statement explains the scientific rationale for having emergency suction equipmentavailable during resuscitation efforts? a. Gastric distention can occur as a result of ventilation. b. It is needed to assist when intubating the client. c. This equipment will ensure a patent airway. d. It keeps the vomitus away from the health-care provider. 4. Which equipment must be immediately brought to the client¶s bedside when a code is calledfor a client who has experienced a cardiac arrest? a. A ventilator. b. A crash cart. c. A gurney. d. Portable oxygen

.5. The nursing administrator responds to a code situation. When assessing the situation, whichrole must the administrator ensure is performed for legal purposes and continuity of care of theclient? a. A person is ventilating with an ambu bag .b. A person is performing chest compressions correctly. c. A person is administering medications as ordered. d. A person is keeping an accurate record of the code. 6. The nurse in the emergency department has admitted five (5) clients in the last two (2) hourswith complaints of fever and gastrointestinal distress. Which question would be most appropriatefor the nurse to ask each client to determine if there is a bioterrorism threat? a. ³Do you work or live near any large power lines?´ b. ³Where were you immediately before you got sick?´ c. ³Can you write down everything you ate today?´ d. ³What other health problems do you have? ´7. The health-care facility has been notified that an alleged inhalation anthrax exposure hasoccurred at the local post office. Which category of personal protective equipment (PPE) wouldthe response team wear? a. Level A b. Level B c. Level C d. Level D 8. The nurse is teaching a class on bioterrorism and is discussing personal protective equipment(PPE). Which statement is the most important fact that must be shared with the participants? a. Health-care facilities should keep masks at entry doors .b. The respondent should be trained in the proper use of PPE. c. No single combination of PPE protects against all hazards. d. The EPA has divided PPE into four levels of protection 9. The nurse is teaching a class on bioterrorism. What is the scientific rationale for designating aspecific area for decontamination?

a. Showers and privacy can be provided to the client in this area. b. This area isolates the clients who have been exposed to the agent .c. It provides a centralized area for stocking the needed supplies. d. It prevents secondary contamination to the health-care providers. 10. The triage nurse in a large trauma center has been notified of an explosion in a majorchemical manufacturing plant. Which action should the nurse implement first when the clientsarrive at the emergency department? a. Triage the clients and send them to the appropriate areas. b. Thoroughly wash the clients with soap and water and then rinse. c. Remove the clients¶ clothing and have them shower. d. Assume the clients have been decontaminated at the plant.

Nursing Board Exam Review Questions in Emergency Part 5/20 (ANSWER KEY) 1. Answer: DRationale: Nurses should protect themselves against possible communicable disease, such asHIV, hepatitis, or any types of sexually transmitted disease. 2. Answer: CRationale: Unexpected death occurring within1 hour of the onset of cardiovascular symptoms isthe definition of sudden cardiac death. 3. Answer: ARationale: Gastric distention occurs from overventilating clients. When compressions areperformed, the pressure will cause vomiting that could be aspirated into the lungs. 4. Answer: BRationale: The crash cart is the mobile unit that has the defibrillator and all the medications andsupplies needed to conduct a code. 5. Answer: DRationale: The chart is a legal document and the code must be documented in the chart andprovide information that may be needed in the intensive care unit. 6. Answer: BRationale: The nurse should take note of any unusual illness for the time of year or clusters ofclients coming from a single geographical location who all exhibit signs/symptoms of possiblebiological terrorism. 7. Answer: ARationale: Level A protection is worn when the highest level of respiratory, skin, eye, and mucous membrane protection is required.In this situation of possible inhalation of anthrax, such protection is required.

8. Answer: CRationale: The health-care providers are not guaranteed absolute protects. The nurse should takenote of any unusual illness for the time of year or clusters of clients coming from a singlegeographical location who all exhibit signs/symptoms of possible biological terrorism.ion, evenwith all the training and protective equipment. 9. Answer: DRationale: Avoiding cross contamination is a priority for personnel and equipment²the fewernumber of people exposed, the safer the community and area. 10. Answer: CRationale: This is the first step. Depending on the type of exposure, this step alone can remove alarge portion of exposure.

Nursing Board Exam Review Questions in Emergency Part 4/20 1. The nurse is planning a program for clients at a health fair regarding the prevention and earlydetection of cancer of the pancreas. Which self-care activity should the nurse teach that is anexample of primary nursing care? a. Monitor for elevated blood glucose at random intervals. b. Inspect the skin and sclera of the eyes for a yellow tint. c. Limit meat in the diet and eat a diet that is low in fats. d. Instruct the client with hyperglycemia about insulin injections. 2. The client diagnosed with cancer of the pancreas is being discharged to start chemotherapy inthe HCP¶s office. Which statement made by the client indicates the client understands thedischarge instructions? a. ³I will have to see the HCP every day for six (6) weeks for my treatments.´ b. ³I should write down all my questions so I can ask them when I see the HCP.´ c. ³I am sure that this is not going to be a serious problem for me to deal with.´ d. ³The nurse will give me an injection in my leg and I will get to go home.´ 3. The nurse caring for a client diagnosed with cancer of the pancreas writes the collaborativeproblem of ³altered nutrition.´ Which intervention should the nurse include in the plan of care? a. Continuous feedings via PEG tube. b. Have the family bring in foods from home.

c. Assess for food preferences. d. Refer to the dietitian. 4. The client is taken to the emergency department with an injury to the left arm. Which actionshould the nurse take first? a. Assess the nail beds for capillary refill time. b. Remove the client¶s clothing from the arm. c. Call radiology for a STAT x-ray of the extremity. d. Prepare the client for the application of a cast. 5. The nurse finds the client unresponsive on the floor of the bathroom. Which action should thenurse implement first? a. Check the client for breathing. b. Assess the carotid artery for a pulse. c. Shake the client and shout. d. Call a code via the bathroom call light. 6. Which behavior by the unlicensed assistive personnel who is performing cardiac compressionson an adult client during a code warrants immediate intervention by the nurse? a. Has one hand on the lower half of the sternum above the xiphoid process. b. Performs cardiac compressions and allows for rescue breathing. c. Depresses the sternum 0.5 to one (1) inch during compressions. d. Requests to be relieved from performing compressions because of exhaustion. 7. Which is the most important intervention for the nurse to implement when participating in acode? a. Elevate the arm after administering medication. b. Maintain sterile technique throughout the code. c. Treat the client¶s signs/symptoms; do not watch the monitor. d. Be sure to provide accurate documentation of what happened in the code. 8. The CPR instructor is explaining what an automated external defibrillator (AED) does tostudents in a CPR class. Which statement best describes an AED?

a. It analyzes the rhythm and shocks the client in ventricular fibrillation. b. The client will be able to have synchronized cardioversion with the AED. c. It will keep the health-care provider informed of the client¶s oxygen level. d. The AED will perform cardiac compressions on the client. 9. The nurse is caring for clients on a medical floor. Which client is most likely to experiencesudden cardiac death? a. The 84-year-old client exhibiting uncontrolled atrial fibrillation. b. The 60-year-old client exhibiting asymptomatic sinus bradycardia. c. The 53-year-old client exhibiting ventricular fibrillation. d. The 65-year-old client exhibiting supraventricular tachycardia. 10. Which health-care team member referral should be made when a code is being conducted ona client in a community hospital? a. The hospital chaplain. b. The social worker. c. The respiratory therapist. d. The director of nurses.

Nursing Board Exam Review Questions in Emergency Part 4/20 (ANSWER KEY)

1. Answer: CRationale: Limiting the intake of meat and fats in the diet would be an example of primaryinterventions. Risk factors for the development of cancer of the pancreas are cigarette smokingand eating a high-fat diet that is high in animal protein. By changing these behaviors the clientcould possibly prevent the development of cancer of the pancreas. Other risk factors includegenetic predisposition and exposure to industrial chemicals. 2. Answer: BRationale: The most important person in the treatment of the cancer is the client. Research hasproved that the more involved a client becomes in his or her care, the better the prognosis.Clients should have a chance to ask all the questions that they have.

3. Answer: DRationale: A collaborative intervention would be to refer to the nutrition expert, the dietitian. 4. Answer: ARationale: The nurse should assess the nail beds for the capillary refill time. A prolonged time(greater than three seconds) indicates impaired circulation to the extremity. 5. Answer: CRationale: This is the first intervention the nurse should implement after finding the clientunresponsive on the floor. 6. Answer: CRationale: The sternum should be depressed 1.5 to 2 inches during compressions to ensureadequate circulation of blood to the body; therefore, the nurse needs to correct the assistant .7. Answer: CRationale: This is the most important intervention.The nurse should always treat the client based on the nurse¶s assessment and data from themonitors; an intervention should not be based on data from the monitors without the nurse¶sassessment. 8. Answer: ARationale: This is the correct statement explaining what an AED does when used in a code. 9. Answer: CRationale: Ventricular fibrillation is the most common dysrhythmia associated with suddencardiac death; ventricular fibrillation is responsible for 65% to 85% of sudden cardiac deaths. 10. Answer: ARationale: The chaplain should be called to help address the client¶s family or significant others.A small community hospital would not have a 24-hour on-duty pastoral service.

Nursing Board Sample Review Questions inEmergency22 Jul, 2010 | Written by Nursingbuzz_editor | under Emergency Nursing Review Questions,Emergency Questions Nursing Board Exam Review Questions in Emergency Part 3/20 1. A client with multiple injury following a vehicular accident is transferred to the critical careunit. He begins to complain of increased abdominal pain in the left upper quadrant. A rupturedspleen is diagnosed and he is scheduled for emergency splenectomy. In preparing the client forsurgery, the nurse should emphasize in his teaching plan the: a. Complete safety of the procedure b. Expectation of postoperative bleeding c. Risk of the procedure with his other injuries d. Presence of abdominal drains for several days after surgery 2. After you managed to stabilize the respiratory function of your burn patient, your next goal isto prevent this you have to replace the lost fluid and electrolytes. In starting fluid replacementtherapy, the

total volume and rate of IV fluid repalcement are gauged by the patient¶s responseand by the patient¶s response and by the resuscitation formula. In determining the adequacy offluid resuscitation, it is essential for you to monitor the: a. urine output b. blood pressure c. intracranial pressure d. cardiac output 3. You are a nurse in the emergency department and it is during the shift that Mr. CT is admittedin the area due to a fractured skull from a motor accident. You scheduled him for surgery underwhich classification? a. Urgent b. Emergent c. Required d. Elective 4. Lucky was in a vehicular acccident where he sustained injury to his left ankle. In theEmergency room, you noticed anxious he looks. You establish rapport with him and to reducehis anxiety, you initially: a. Identify yourself and state your purpose in being with the client b. Take him to the radiology section for x-ray of affected extremity c. Talk to the physician for an order of valium d. Do inspection and palpation to check extent of his injuries 5. The client diagnosed with a mild concussion is being discharged from the emergencydepartment. Which discharge instruction should the nurse teach the client¶s significant other? a. Awaken the client every two hours. b. Monitor for increased intracranial pressure. c. Observe frequently for hypervigilance. d. Offer the client food every three to four hours. 6. The client diagnosed with Addison¶s disease is admitted to the emergency department after aday at the lake. The client is lethargic, forgetful, and weak. Which intervention should be theemergency department nurse¶s first action?

a. Start an IV with an 18-gauge needle and infuse NS rapidly .b. Have the client wait in the waiting room until a bed is available .c. Perform a complete head-to-toe assessment. d. Collect urinalysis and blood samples for a CBC and calcium level. 7. The nurse caring for a client diagnosed with cancer of the pancreas writes the nursingdiagnosis of ³risk for altered skin integrity related to pruritus.´ Which interventions should thenurse implement? a. Assess tissue turgor. b. Apply antifungal creams. c. Monitor bony prominences for breakdown. d. Have the client keep the fingernails short. 8. The client diagnosed with cancer of the head of the pancreas is two (2) dayspostpancreatoduodenectomy (Whipple¶s procedure). Which nursing problem has the highestpriority? a. Anticipatory grieving. b. Fluid volume imbalance. c. Acute incisional pain. d. Altered nutrition. 9. The client is diagnosed with cancer of the head of the pancreas. When assessing the patient,which signs and symptoms would the nurse expect to find? a. Clay-colored stools and dark urine .b. Night sweats and fever. c. Left lower abdominal cramps and tenesmus. d. Nausea and coffee-ground emesis. 10. The client admitted to rule out pancreatic islet tumors complains of feeling weak, shaky, andsweaty. Which should be the first intervention implemented by the nurse? a. Start an IV with D5W. b. Notify the health-care provider.

c. Perform a bedside glucose check. d. Give the client some orange juice.

Nursing Board Exam Review Questions in Emergency Part 3/20 (ANSWER KEY) 1. Answer: DRationale: Presence of abdominal drains for several days after surgeryDrains are usually inserted into the splenic bed to facilitate removal of fluid in the area that couldlead to abscess formation. 2. Answer: ARationale: to establish the sufficiency of fluid resuscitation, urine output totals an index of renalperfusion. Urine output totals an index of renal perfusion, urine output totals of 30-50 ml/hourhave been used as resuscitation goals. Other indicators of adequate fluid replacement are systolicblood pressure exceeding 100 mmHg, a pulse rate less than110 beats/min or both. 3. Answer: BRationale: Emergent surgery is performed, immediately without delay to maintain life, limb ororgan, remove damage and stop bleeding. Urgent surgery requires prompt attention and is donefew hours but within 24 to 48 hours. Required surgery is done within a few weeks as surgery isimportant. Elective surgery is scheduled and done at the convenience of client as failure to havesurgery is not catastrophic. Optional surgeries are done by preference only. 4. Answer: ARationale: Introducing self initiates the nurse-patient interaction, relationship and the purpose ofbeing with the client. This prevents confusion and let the client know what to expect, therebyreducing anxiety .5. Answer: ARationale: Awakening the client every 2 hours allows the identification of headache, dizziness,lethargy, irritability, and anxiety²all signs of post-concussion syndrome²that would warrantthe significant other¶s taking the client back to the emergency department. 6. Answer: ARationale: This client has been exposed to wind and sun at the lake during the hours prior tobeing admitted to the emergency department. This predisposes the client to dehydration and anAddisonian crisis. Rapid IV fluid replacement is necessary .7. Answer: DRationale: Keeping the fingernails short will reduce the chance of breaks in the skin fromscratching. 8. Answer: BRationale: This is a major abdominal surgery, and there are massive fluid volume shifts thatoccur when this type of trauma is experienced by the body. Maintaining the circulatory systemwithout overloading it requires extremely close monitoring. 9. Answer: ARationale: The client will have jaundice, clay-colored stools, and tea-colored urine resulting fromblockage of the bile drainage

.10. Answer: CRationale: These are symptoms of an insulin reaction (hypoglycemia). A bedside glucose checkshould be done. Pancreatic islet tumors can produce hyperinsulinemia or hypoglycemia.

Nursing Board Exam Review Questions in Emergency Part 2/20 1 Which nursing intervention would be appropriate when caring for a client who has sustained anelectrical burn? a. Applying ice to the burned area b. Flushing the burn area with large amounts of water c. Monitoring the client with cardiac telemetryd . Preparing to administer the chemical antidote 2. Eddie, 40 years old, is brought to the emergency room after the crash of his private plane. Hehas suffered multiple crushing wounds of the chest, abdomen and legs. It is feared his leg mayhave to be amputated.When Eddie arrives in the emergency room, the assessment that assume the greatest priority are: a. Level of consciousness and pupil size b. Abdominal contusions and other wounds c. Pain, Respiratory rate and blood pressure d. Quality of respirations and presence of pulses. 3. An emergency treatment for an acute asthmatic attack is Adrenaline 1:1000 givenhypodermically. This is given to: a. increase BP b. decrease mucosal swelling c. relax the bronchial smooth muscle d. decrease bronchial secretions 4. Intervention for a pt. who has swallowed a Muriatic Acid includes all of the following except a. administering an irritant that will stimulate vomiting b. aspirating secretions from the pharynx if respirations are affected c. neutralizing the chemical

d. washing the esophagus with large volumes of water via gastric lavage 5. John, 16 years old, is brought to the ER after a vehicular accident. He is pronounced dead onarrival. When his parents arrive at the hospital, the nurse should: a. ask them to stay in the waiting area until she can spend time alone with them b. speak to both parents together and encourage them to support each other and express theiremotions freely c. Speak to one parent at a time so that each can ventilate feelings of loss without upsetting the other d. ask the MD to medicate the parents so they can stay calm to deal with their son¶s death .6. A nurse is eating in the hospital cafeteria when a toddler at a nearby table chokes on a piece offood and appears slightly blue. The appropriate initial action should be to a. Begin mouth to mouth resuscitation b. Give the child water to help in swallowing c. Perform 5 abdominal thrusts d. Call for the emergency response team 7. A client is admitted from the emergency department with severe-pain and edema in the rightfoot. His diagnosis is gouty arthritis. When developing a plan of care, which action would havethe highest priority? a. Apply hot compresses to the affected joints. b. Stress the importance of maintaining good posture to prevent deformities .c. Administer salicylates to minimize the inflammatory reaction. d. Ensure an intake of at least 3000 ml of fluid per day. 8. The Heimlich maneuver (abdominal thrust), for acute airway obstruction, attempts to :a. Force air out of the lungs b. Increase systemic circulation c. Induce emptying of the stomach d. Put pressure on the apex of the heart 9. A nurse is performing CPR on an adult patient. When performing chest compressions, thenurse understands the correct hand placement is located over the

a. upper half of the sternum b. upper third of the sternum c. lower half of the sternum d. lower third of the sternum John, 16 years old, is brought to the ER after a vehicular accident. He is pronounced dead onarrival. When his parents arrive at the hospital, the nurse should: a. ask them to stay in the waiting area until she can spend time alone with them b. speak to both parents together and encourage them to support each other and express theiremotions freely c. Speak to one parent at a time so that each can ventilate feelings of loss without upsetting theother d. ask the MD to medicate the parents so they can stay calm to deal with their son¶s death.

Nursing Board Exam Review Questions in Emergency Part 2/20 (ANSWER KEY) 1. Answer: CRationale: Because of the effects of the electrical current on the cardiovascular system, all clientsexperiencing electrical burns should be placed on a cardiac monitor. Applying ice is inappropriate for any type of burn. Only chemical burns should be flushed with large amounts ofwater. Chemical antidotes may be used for chemical burns for which an antidote has beenidentified. 2. Answer: DRationale: Respiratory and cardiovascular functions are essential for oxygenation. These are toppriorities to trauma management. Basic life functions must be maintained or reestablished 3. Answer: CRationale: Acute asthmatic attack is characterized by severe bronchospasm which can berelieved by the immediate administration of bronchodilators. Adrenaline or Epinephrine is anadrenergic agent that causes bronchial dilation by relaxing the bronchial smooth muscles. 4. Answer: ARationale: Swallowing of corrosive substances causes severe irritation and tissue destruction ofthe mucous membrane of the GI tract. Measures are taken to immediately remove the toxin orreduce its absorption. For corrosive poison ingestion, such as in muriatic acid where burn orperforation of the mucosa may occur, gastric emptying procedure is immediately instituted, Thisincludes gastric lavage and the administration of activated charcoal to absorb the poison.Administering an irritant with the concomitant vomiting to remove the swallowed poison willfurther cause irritation and damage to the mucosal lining of the digestive tract. Vomiting is onlyindicated when non-corrosive poison is swallowed. 5. Answer: BRationale: Sudden death of a family member creates a state of shock on the family. They go intoa stage of denial and anger in their grieving. Assisting them with information they need to

know,answering their questions and listening to them will provide the needed support for them to moveon and be of support to one another. 6. Answer: CRationale: Perform 5 abdominal thrusts. At this age, the most effective way to clear the airway offood is to perform abdominal thrusts. 7. Answer: DRationale: Ensure an intake of at least 3000 ml of fluid per day. Gouty arthritis is a metabolicdisease marked by urate deposits that cause painful arthritic joints. The patient should be urgedto increase his fluid intake to prevent the development of urinary uric acid stones. 8. Answer: ARationale: The Heimlich maneuver is used to assist a person choking on a foreign object. Thepressure from the thrusts lifts the diaphragm, forces air out of the lungs and creates an artificialcough that expels the aspirated material. 9. Answer: CRationale: The exact and safe location to do cardiac compression is the lower half of the15. sternum. Doing it at the lower third of the sternum may cause gastric compression which canlead to a possible aspiration. 10. Answer: BRationale: Sudden death of a family member creates a state of shock on the family. They go intoa stage of denial and anger in their grieving. Assisting them with information they need to know,answering their questions and listening to them will provide the needed support for them to moveon and be of support to one another.

1. You are the charge nurse in an emergency department (ED) and must assign two staff members to cover the triage area. Which team is the most appropriate for this assignment? a. An advanced practice nurse and an experienced LPN/LVN b. An experienced LPN/LVN and an inexperienced RN c. An experienced RN and an inexperienced RN d. An experienced RN and a nursing assistant 2. You are working in the triage area of an ED, and four patients approach the triage desk at the same time. List the order in which you will assess these patients. a. An ambulatory, dazed 25-year-old male with a bandaged head wound b. An irritable infant with a fever, petechiae, and nuchal rigidity c. A 35-year-old jogger with a twisted ankle, having pedal pulse and no deformity d. A 50-year-old female with moderate abdominal pain and occasional vomiting _____, _____, _____, _____

3. In conducting a primary survey on a trauma patient, which of the following is considered one of the priority elements of the primary survey? a. Complete set of vital signs b. Palpation and auscultation of the abdomen c. Brief neurologic assessment d. Initiation of pulse oximetry 4. A 56-year-old patient presents in triage with left-sided chest pain, diaphoresis, and dizziness. This patient should be prioritized into which category? a. High urgent b. Urgent c. Non-urgent d. Emergent 5. The physician has ordered cooling measures for a child with fever who is likely to be discharged when the temperature comes down. Which of the following would be appropriate to delegate to the nursing assistant? a. Assist the child to remove outer clothing. b. Advise the parent to use acetaminophen instead of aspirin. c. Explain the need for cool fluids. d. Prepare and administer a tepid bath. 6. It is the summer season, and patients with signs and symptoms of heat-related illness present in the ED. Which patient needs attention first? a. An elderly person complains of dizziness and syncope after standing in the sun for several hours to view a parade b. A marathon runner complains of severe leg cramps and nausea. Tachycardia, diaphoresis, pallor, and weakness are observed. c. A previously healthy homemaker reports broken air conditioner for days. Tachypnea, hypotension, fatigue, and profuse diaphoresis are observed. d. A homeless person, poor historian, presents with altered mental status, poor muscle coordination, and hot, dry, ashen skin. Duration of exposure is unknown. 7. You respond to a call for help from the ED waiting room. There is an elderly patient lying on the floor. List the order for the actions that you must perform. a. Perform the chin lift or jaw thrust maneuver. b. Establish unresponsiveness. c. Initiate cardiopulmonary resuscitation (CPR). d. Call for help and activate the code team. e. Instruct a nursing assistant to get the crash cart. _____, _____, _____, _____, _____ 8. The emergency medical service (EMS) has transported a patient with severe chest pain.

As the patient is being transferred to the emergency stretcher, you note unresponsiveness, cessation of breathing, and no palpable pulse. Which task is appropriate to delegate to the nursing assistant? a. Chest compressions b. Bag-valve mask ventilation c. Assisting with oral intubation d. Placing the defibrillator pads 9. An anxious 24-year-old college student complains of tingling sensations, palpitations, and chest tightness. Deep, rapid breathing and carpal spasms are noted. What priority nursing action should you take? a. Notify the physician immediately. b. Administer supplemental oxygen. c. Have the student breathe into a paper bag. d. Obtain an order for an anxiolytic medication. 10.An experienced traveling nurse has been assigned to work in the ED; however, this is the nurse’s first week on the job. Which area of the ED is the most appropriate assignment for the nurse? a. Trauma team b. Triage c. Ambulatory or fats track clinic d. Pediatric medicine team 11.A tearful parent brings a child to the ED for taking an unknown amount of children’s chewable vitamins at an unknown time. The child is currently alert and asymptomatic. What information should be immediately reported to the physician? a. The ingested children’s chewable vitamins contain iron. b. The child has been treated several times for ingestion of toxic substances. c. The child has been treated several times for accidental injuries. d. The child was nauseated and vomited once at home. 12.In caring for a victim of sexual assault, which task is most appropriate for an LPN/LVN? a. Assess immediate emotional state and physical injuries b. Collect hair samples, saliva swabs, and scrapings beneath fingernails. c. Provide emotional support and supportive communication. d. Ensure that the “chain of custody” is maintained. 13.You are caring for a victim of frostbite to the feet. Place the following interventions in the correct order.

a. Apply a loose, sterile, bulky dressing. b. Give pain medication. c. Remove the victim from the cold environment. d. Immerse the feet in warm water 100o F to 105o F (40.6o C to 46.1o C) _____, _____, _____, _____ 14.A patient sustains an amputation of the first and second digits in a chainsaw accident. Which task should be delegated to the LPN/LVN? a. Gently cleanse the amputated digits with Betadine solution. b. Place the amputated digits directly into ice slurry. c. Wrap the amputated digits in sterile gauze moistened with saline. d. Store the amputated digits in a solution of sterile normal saline. 15.A 36-year-old patient with a history of seizures and medication compliance of phenytoin (Dilantin) and carbamazepine (Tegretol) is brought to the ED by the MS personnel for repetitive seizure activity that started 45 minutes prior to arrival. You anticipate that the physician will order which drug for status epilepticus? a. PO phenytoin and carbamazepine b. IV lorazepam (Ativan) c. IV carbamazepam d. IV magnesium sulfate 16.You are preparing a child for IV conscious sedation prior to repair of a facial laceration. What information should you immediately report to the physician? a. The parent is unsure about the child’s tetanus immunization status. b. The child is upset and pulls out the IV. c. The parent declines the IV conscious sedation. d. The parent wants information about the IV conscious sedation. 17.An intoxicated patient presents with slurred speech, mild confusion, and uncooperative behavior. The patient is a poor historian but admits to “drinking a few on the weekend.” What is the priority nursing action for this patient? a. Obtain an order for a blood alcohol level. b. Contact the family to obtain additional history and baseline information. c. Administer naloxone (Narcan) 2 - 4 mg as ordered. d. Administer IV fluid support with supplemental thiamine as ordered. 18.When an unexpected death occurs in the ED, which of the following tasks is most appropriate to delegate to the nursing assistant? a. Escort the family to a place of privacy. b. Go with the organ donor specialist to talk to the family.

c. Assist with postmortem care. d. Assist the family to collect belongings. 19.Following emergency endotracheal intubation, you must verify tube placement and secure the tube. List in order the steps that are required to perform this function? a. Obtain an order for a chest x-ray to document tube placement. b. Secure the tube in place. c. Auscultate the chest during assisted ventilation. d. Confirm that the breath sounds are equal and bilateral. _____, _____, _____, _____ 20.A teenager arrives by private car. He is alert and ambulatory, but this shirt and pants are covered with blood. He and his hysterical friends are yelling and trying to explain that that they were goofing around and he got poked in the abdomen with a stick. Which of the following comments should be given first consideration? a. “There was a lot of blood and we used three bandages.” b. “He pulled the stick out, just now, because it was hurting him.” c. “The stick was really dirty and covered with mud.” d. “He’s a diabetic, so he needs attention right away.” 21.A prisoner, with a known history of alcohol abuse, has been in police custody for 48 hours. Initially, anxiety, sweating, and tremors were noted. Now, disorientation, hallucination, and hyper-reactivity are observed. The medical diagnosis is delirium tremens. What is the priority nursing diagnosis? a. Risk for Injury related to seizures b. Risk for Other-Directed Violence related to hallucinations c. Risk for Situational Low Self-esteem related to police custody d. Risk for Nutritional Deficit related to chronic alcohol abuse 22.You are assigned to telephone triage. A patient who was stung by a common honey bee calls for advice, reports pain and localized swelling, but denies any respiratory distress or other systemic signs of anaphylaxis. What is the action that you should direct the caller to perform? a. Call 911. b. Remove the stinger by scraping. c. Apply a cool compress. d. Take an oral antihistamine. 23.In relation to submersion injuries, which task is most appropriate to delegate to an LPN/LVN?

a. Talk to a community group about water safety issues. b. Stabilize the cervical spine for an unconscious drowning victim. c. Remove wet clothing and cover the victim with a warm blanket. d. Monitor an asymptomatic near-drowning victim. 24.You are assessing a patient who has sustained a cat bite to the left hand. The cat is upto-date immunizations. The date of the patient’s last tetanus shot is unknown. Which of the following is the priority nursing diagnosis? a. Risk for Infection related to organisms specific to cat bites b. Impaired Skin Integrity related to puncture wounds c. Ineffective Health Maintenance related to immunization status d. Risk for Impaired Mobility related to potential tendon damage 25.These patients present to the ED complaining of acute abdominal pain. Prioritize them in order of severity. a. A 35-year-old male complaining of severe, intermittent cramps with three episodes of watery diarrhea, 2 hours after eating b. A 11-year-old boy with a low-grade fever, left lower quadrant tenderness, nausea, and anorexia for the past 2 days c. A 40-year-old female with moderate left upper quadrant pain, vomiting small amounts of yellow bile, and worsening symptoms over the past week d. A 56-year-old male with a pulsating abdominal mass and sudden onset of pressure-like pain in the abdomen and flank within the past hour _____, _____, _____, _____ 26.The nursing manager decides to form a committee to address the issue of violence against ED personnel. Which combination of employees is best suited to fulfill this assignment? a. ED physicians and charge nurses b. Experienced RNs and experienced paramedics c. RNs, LPN/LVNs, and nursing assistants d. At least one representative from each group of ED personnel 27.In a multiple-trauma victim, which assessment finding signals the most serious and lifethreatening condition? a. A deviated trachea b. Gross deformity in a lower extremity c. Decreased bowel sounds d. Hematuria 28.A patient in a one-car rollover presents with multiple injuries. Prioritize the interventions that must be initiated for this patient.

a. Secure/start two large-bore IVs with normal saline b. Use the chin lift or jaw thrust method to open the airway. c. Assess for spontaneous respirations d. Give supplemental oxygen per mask. e. Obtain a full set of vital signs. f. Remove patient’s clothing. g. Insert a Foley catheter if not contraindicated. _____, _____, _____, _____, ____, ____, ____ 29.In the work setting, what is your primary responsibility in preparing for disaster management that includes natural disasters or bioterrorism incidents? a. Knowledge of the agency’s emergency response plan b. Awareness of the signs and symptoms for potential agnets of bioterrorism c. Knowledge of how and what to report to the CDC d. Ethical decision-making about exposing self to potentially lethal substances 30.You are giving discharge instructions to a woman who has been treated for contusions and bruises sustained during an episode of domestic violence. What is your priority intervention for this patient? a. Transportation arrangements to a safe house b. Referral to a counselor c. Advise about contacting the police d. Follow-up appointment for injuries

RATIONALE MEDICAL – SURGICAL EMERGENCIES 1. ANSWER C – Triage requires at least one experienced RN. Pairing an experienced RN with inexperienced RN provides opportunities for mentoring. Advanced practice nurses are qualified to perform triage; however, their services are usually required in other areas of the ED. An LPN/LVN is not qualified to perform the initial patient assessment or decision making. Pairing an experienced RN with a nursing assistant is the second best option, because the assistant can obtain vital signs and assist in transporting. 2. ANSWER B, A, D, C – An irritable infant with fever and petechiae should be further assessed for other meningeal signs. The patient with the head wound needs additional history and assessment for intracranial pressure. The patient with moderate abdominal pain is uncomfortable, but not unstable at this point. For the ankle injury, medical evaluation can be delayed 24 – 48 hours if necessary.

3. ANSWER C – A brief neurologic assessment to determine level of consciousness and pupil reaction is part of the primary survey. Vital signs, assessment of the abdomen, and initiation of pulse oximetry are considered part of the secondary survey. 4. ANSWER D – Chest pain is considered an emergent priority, which is defined as potentially life-threatening. Patients with urgent priority need treatment within 2 hours of triage (e.g. kidney stones). Non-urgent conditions can wait for hours or even days. (High urgent is not commonly used; however, in 5-tier triage systems, High urgent patients fall between emergent and urgent in terms of the time lapsing prior to treatment). 5. ANSWER A – The nursing assistant can assist with the removal of the outer clothing, which allows the heat to dissipate from the child’s skin. Advising and explaining are teaching functions that are the responsibility of the RN. Tepid baths are not usually performed because of potential for rebound and shivering. 6. ANSWER D – The homeless person has symptoms of heat stroke, a medical emergency, which increases risk for brain damage. Elderly patients are at risk for heat syncope and should be educated to rest in cool area and avoid future similar situations. The runner is having heat crams, which can be managed with rest and fluids. The housewife is experiencing heat exhaustion, and management includes fluids (IV or parenteral) and cooling measures. The prognosis for recovery is good. 7. ANSWER B, D, A, C, E – Establish unresponsiveness first. (The patient may have fallen and sustained a minor injury.) If the patient is unresponsive, get help and have someone initiate the code. Performing the chin lift or jaw thrust maneuver opens the airway. The nurse is then responsible for starting CPR. CPR should not be interrupted until the patient recovers or it is determined that heroic efforts have been exhausted. A crash cart should be at the site when the code team arrives; however, basic CPR can be effectively performed until the team arrives. 8. ANSWER A – Nursing assistants are trained in basic cardiac life support and can perform chest compressions. The use of the bag-valve mask requires practice and usually a respiratory therapist will perform this function. The nurse or the respiratory therapist should provide PRN assistance during intubation. The defibrillator pads are clearly marked; however, placement should be done by the RN or physician because of the potential for skin damage and electrical arcing. 9. ANSWER C – The patient is hyperventilating secondary to anxiety, and breathing into a paper bag will allow rebreathing of carbon dioxide. Also, encouraging slow breathing will help. Other treatments such as oxygen and medication may be needed if other causes are identified. 10. ANSWER C – The fast track clinic will deal with relatively stable patients. Triage, trauma, and pediatric medicine should be staffed with experienced nurses who know the hospital routines and policies and can rapidly locate equipment.

11. ANSWER A – Iron is a toxic substance that can lead to massive hemorrhage, coma, shock, and hepatic failure. Deferoxame is an antidote that can be used for severe cases of iron poisoning. Other information needs additional investigation, but will not change the immediate diagnostic testing or treatment plan. 12. ANSWER C – The LPN/LVN is able to listen and provide emotional support for her patients. The other tasks are the responsibility of an RN or, if available, a SANE (sexual assault nurse examiner) who has received training to assess, collect and safeguard evidence, and care for these victims. 13. ANSWER C, B, D, A – The victim should be removed from the cold environment first, and then the rewarming process can be initiated. It will be painful, so give pain medication prior to immersing the feet in warmed water. 14. ANSWER C – The only correct intervention is C. the digits should be gently cleansed with normal saline, wrapped in sterile gauze moistened with saline, and placed in a plastic bag or container. The container is then placed on ice. 15. ANSWER B – IV Lorazepam (Ativan) is the drug of choice for status epilepticus. Tegretol is used in the management of generalized tonic-clonic, absence or mixed type seizures, but it does not come in an IV form. PO (per os) medications are inappropriate for this emergency situation. Magnesium sulfate is given to control seizures in toxemia of pregnancy. 16. ANSWER C – Parent refusal is an absolute contraindication; therefore, the physician must be notified. Tetanus status can be addressed later. The RN can restart the IV and provide information about conscious sedation; if the parent still notsatisfied, the physician can give more information. 17. ANSWER D – The patient presents with symptoms of alcohol abuse and there is a risk for Wernicke’s syndrome, which is caused by a thiamine deficiency. Multiples drug abuse is not uncommon; however, there is nothing in the question that suggests an opiate overdose that requires naloxone. Additional information or the results of the blood alcohol level are part of the total treatment plan but should not delay the immediate treatment. 18. ANSWER C – Postmortem care requires some turning, cleaning, lifting, etc., and the nursing assistant is able to assist with these duties. The RN should take responsibility for the other tasks to help the family begin the grieving process. In cases of questionable death, belongings may be retained for evidence, so the chain of custody would have to be maintained. 19. ANSWER C, D, B, A – Auscultating and confirming equal bilateral breath sounds should be performed in rapid succession. If the sounds are not equal or if the sounds are heard over the mid-epigastric area, tube placement must be corrected immediately. Securing the tube is appropriate while waiting for the x-ray study.

20. ANSWER B – An impaled object may be providing a tamponade effect, and removal can precipitate sudden hemodynamic decompensation. Additional history including a more definitive description of the blood loss, depth of penetration, and medical history should be obtained. Other information, such as the dirt on the stick or history of diabetes, is important in the overall treatment plan, but can be addressed later. 21. ANSWER A – The patient demonstrates neurologic hyperactivity and is on the verge of a seizure. Patient safety is the priority. The patient needs chlordiazepoxide (Librium) to decrease neurologic irritability and phenytoin (Dilantin) for seizures. Thiamine and haloperidol (Haldol) will also be ordered to address the other problems. The other diagnoses are pertinent but not as immediate. 22. ANSWER B – The stinger will continue to release venom into the skin, so prompt removal of the stinger is advised. Cool compresses and antihistamines can follow. The caller should be further advised about symptoms that require 911 assistance. 23. ANSWER D – The asymptomatic patient is currently stable but should be observed for delayed pulmonary edema, cerebral edema, or pneumonia. Teaching and care of critical patients is an RN responsibility. Removing clothing can be delegated to a nursing assistant. 24. ANSWER A – Cat’s mouths contain a virulent organism, Pasteurella multocida, that can lead to septic arthritis or bacteremia. There is also a risk for tendon damage due to deep puncture wounds. These wounds are usually not sutured. A tetanus shot can be given before discharge. 25. ANSWER D, B, C, A – The patient with a pulsating mass has an abdominal aneurysm that may rupture and he may decompensate suddenly. The 11-year-old boy needs evaluation to rule out appendicitis. The woman needs evaluation for gallbladder problems that appear to be worsening. The 35-year-old man has food poisoning, which is usually selflimiting. 26. ANSWER D – At least one representative from each group should be included because all employees are potential targets fro violence in the ED. 27. ANSWER A – A deviated trachea is a symptoms of tension pneumothorax. All of the other symptoms need to be addressed, but are of lesser priority. 28. ANSWER C, B, D, A, E, F, G – For a multiple trauma victim, many interventions will occur simultaneously as team members assist in the resuscitation. Methods to open the airway such as the chin lift or jaw thrust can be used simultaneously while assessing for spontaneous respirations. However, airway and oxygenation are priority. Starting IVs for fluid resuscitation is part of supporting circulation. (EMS will usually establish at least one IV in the field.) Nursing assistants can be directed to take vitals and remove clothing. Foley catheter is necessary to closely monitor output. 29. ANSWER A – In preparing for disasters, the RN should be aware of the emergency

response plan. The plan gives guidance that includes roles of team members, responsibilities, and mechanisms of reporting. Signs and symptoms of many agents will mimic common complaints, such as flu-like symptoms. Discussions with colleagues and supervisors may help the individual nurse to sort through ethical dilemmas related to potential danger to self. 30. ANSWER A – Safety is a priority for this patient, and she should not return to a place where violence could reoccur. The other options are important for the long term management of this care.

Terms

Definitions

Observe the patient's respiratory effort.

During the primary assessment of a trauma victim, the nurse determines

rationale:

that the patient is breathing and has

Even with a patent airway, patients can

an unobstructed airway. Which action

have other problems that compromise

should the nurse take next?

ventilation, so the next action is to assess the patient's breathing. The other actions also are part of the initial survey but assessment of breathing should be done immediately after assessing for airway patency. Initiate isotonic fluid infusion through two

During the primary survey of a

large-bore IV lines.

patient with multiple traumatic injuries, the nurse observes that the

rationale:

patient's right pedal pulses are

The assessment data indicate that the

absent and the leg is swollen. Which

patient may have arterial trauma and

of these actions will the nurse take

hemorrhage. When a possibly life-

next?

threatening injury is found during the primary survey, the nurse should immediately start interventions before proceeding with the survey. Although a CBC is indicated, administration of IV fluids should be started first. Completion of the primary survey and further assessment should be completed after the IV fluids are initiated. Rapidly infuse cold normal saline.

After resuscitation, a patient who had a cardiac arrest is nonresponsive

rationale:

to commands and therapeutic

When therapeutic hypothermia is used

hypothermia is prescribed. Which

postresuscitation, cold normal saline is

action will the nurse include in the

infused to rapidly lower body temperature

plan of care?

to 89.6° F to 93.2° F (32° C to 34° C). Since hypothermia will decrease brain activity, neurologic assessment every 30 minutes is not needed. Sedative medications are administered during therapeutic hypothermia. obtain a Glasgow Coma Scale score.

A patient who is unconscious after a fall from a ladder is transported to

rationale:

the emergency department by family

The Glasgow Coma Scale is included when

members. During the primary survey

assessing for disability during the primary

of the patient, the nurse should

survey. The other information is part of the

_____________

secondary survey. TIG and tetanus-diphtheria toxoid and

An 18-year-old is brought to the

pertussis vaccine (Tdap).

emergency department (ED) with multiple lacerations and tissue

rationale:

avulsion of the right hand. When

For an adult with no previous tetanus

asked about tetanus immunization,

immunizations, TIG and Tdap are

the patient denies having any

recommended. The other immunizations

previous vaccinations. The nurse will

are not sufficient for this patient.

anticipate administration of ______________

ultrasonography.

A patient who has experienced blunt abdominal trauma during a car

rationale:

accident is complaining of increasing

For patients who are at risk for

abdominal pain. The nurse will plan

intraabdominal bleeding, focused

to teach the patient about the

abdominal ultrasonography is the preferred

purpose of ______________

method to assess for intraperitoneal bleeding. An MRI would not be used. Peritoneal lavage is an alternative, but it is more invasive. An NG tube would not be helpful in diagnosis of intraabdominal bleeding.

"I should have sports drinks when

A patient with hypotension and

exercising outside in hot weather."

temperature elevation after doing yard work on a hot day is treated in

rationale:

the ED. After the nurse has

Electrolyte solutions such as sports drinks

completed discharge teaching, which

help replace fluid and electrolytes lost

statement by the patient indicates

when exercising in hot weather. Salt tablets

that the teaching has been effective?

are not recommended because of the risks of gastric irritation and hypernatremia. Antipyretic medications are not effective in lowering body temperature elevations caused by excessive exposure to heat. A patient who is confused is likely to have more severe hyperthermia and will be unable to remember to take appropriate action. attach a cardiac monitor.

When preparing to rewarm a patient with hypothermia, the nurse will plan

rationale:

to _____________

Rewarming can produce dysrhythmias, so the patient should be monitored and treated if necessary. Urinary catheterization and endotracheal intubation are not needed for rewarming. Sympathomimetic drugs tend to stimulate the heart and increase the risk for fatal dysrhythmias such as ventricular fibrillation. Auscultate breath sounds.

A patient who experienced a near drowning accident in a local lake, but

rationale:

now is awake and breathing

Since pulmonary edema is a common

spontaneously, is admitted for

complication after near drowning, the

observation. Which action will be

nurse should assess the breath sounds

most important for the nurse to take

frequently. The other information also will

during the observation period?

be collected by the nurse, but it is not as pertinent to the patient's admission diagnosis. vaccine.

When planning the response to the

potential use of smallpox as an agent rationale:

of terrorism, the emergency

Smallpox infection can be prevented or

department (ED) nurse-manager will

ameliorated by the administration of

plan to obtain sufficient quantities of

vaccine given rapidly after exposure. The

______________

other interventions would be helpful for other agents of terrorism but not for smallpox. The core temperature is 94° F (34.4° C).

When rewarming a patient who arrived in the emergency department

rationale:

(ED) with a temperature of 87° F,

A core temperature of 89.6° F to 93.2° F

which assessment indicates that the

(32° C to 34° C) indicates that sufficient

nurse should discontinue the

rewarming has occurred. Dysrhythmias,

rewarming?

hypotension, and shivering may occur during rewarming and should be treated but are not an indication to stop rewarming the patient. "Is someone at home hurting you?"

When assessing a patient admitted to the emergency department (ED) with

rationale:

a broken arm and facial bruises, the

The nurse's initial response should be to

nurse notes multiple additional

further assess the patient's situation.

bruises in various stages of healing.

Telling the patient not to return home may

Which statement or question by the

be an option once further assessment is

nurse is most appropriate?

done. The patient, not the nurse, is responsible for reporting the abuse. A social worker may be appropriate once further assessment is completed. Give N-acetylcysteine (Mucomyst).

A patient arrives in the emergency department (ED) a few hours after

rationale:

taking "20 to 30" acetaminophen

N-acetylcysteine is the recommended

(Tylenol) tablets. Which action will

treatment to prevent liver damage after

the nurse plan to take?

acetaminophen overdose. The other actions might be used for other types of poisoning, but they will not be appropriate for a patient with acetaminophen poisoning.

assess the patient's current vital signs.

A triage nurse in a busy emergency department assesses a patient who

rationale:

complains of 6/10 abdominal pain

The patient's pain and statement about an

and states, "I had a temperature of

elevated temperature indicate that the

104.6º F (40.3º C) at home." The

nurse should obtain vital signs before

nurse's first action should be to

deciding how rapidly the patient should be

______________

seen by the health care provider. A urinalysis may be needed, but vital signs will provide the nurse with the data needed to determine this. The health care provider will not order a medication before assessing the patient. A patient with a sucking chest wound

The emergency department (ED) triage nurse is assessing four victims

rationale:

of an automobile accident. Which

Most immediate deaths from trauma occur

patient has the highest priority for

because of problems with ventilation, so

treatment?

the patient with a sucking chest wound should be treated first. Face and head fractures can obstruct the airway, but the patient with facial injuries has lacerations only. The other two patients also need rapid intervention but do not have airway or breathing problems. Remove the patient's rings.

The following actions are part of the routine emergency department (ED)

rationale:

protocol for a patient who has been

The patient's rings should be removed first

admitted with multiple bee stings to

because it might not be possible to remove

the hands. Which action should the

them if swelling develops. The other orders

nurse take first?

also should be implemented as rapidly as possible after the nurse has removed the jewelry. Assist with intubation of the patient.

Gastric lavage and administration of activated charcoal are prescribed for

rationale:

an unconscious patient who has been

In an unresponsive patient, intubation is

admitted to the emergency

done before gastric lavage and activated

department (ED) after ingesting 30

charcoal administration to prevent

diazepam (Valium) tablets. Which

aspiration. The other actions will be

action will the nurse plan to take

implemented after intubation.

first?

Place the patient in a shower.

A patient arrives in the emergency department after exposure to

rationale:

radioactive dust. Which action should

The initial action should be to protect staff

the nurse take first?

members and decrease the patient's exposure to the radioactive agent by decontamination. The other actions can be done after the decontamination is completed. apply wet sheets and a fan to the patient.

An unresponsive 78-year-old is admitted to the emergency

rationale:

department (ED) during a summer

The priority intervention is to cool the

heat wave. The patient's core

patient. Antipyretics are not effective in

temperature is 106.2° F (41.2° C),

decreasing temperature in heat stroke, and

blood pressure (BP) 86/52, and pulse

100% oxygen should be given, which

102. The nurse initially will plan to

requires a high flow rate through a non-

_____________

rebreather mask. An older patient would be at risk for developing complications such as pulmonary edema if given fluids at 1000 mL/hr. Lung sounds

When a patient is admitted to the emergency department after a

rationale:

submersion injury, which assessment

The priority assessment data are how well

will the nurse obtain first?

the patient is oxygenating, so lung sounds should be assessed first. The other data also will be collected rapidly but are not as essential as the lung sounds. A patient with a red tag

Following an earthquake, patients are triaged by emergency medical

rationale:

personnel and are transported to the

The red tag indicates a patient with a life-

hospital. Which of these patients will

threatening injury requiring rapid

the nurse need to assess first?

treatment. The other tags indicate patients with less urgent injuries or those who are likely to die. Ask the family members about whether

A patient's family members are in the

they would prefer to remain in the patient

patient room when the patient has a

room or wait outside the room.

cardiac arrest and emergency personnel start resuscitation

rationale:

measures. Which action is best for

Although many family members and

the nurse to take initially?

patients report benefits from family presence during resuscitation efforts, the nurse's initial action should be to determine the preference of these family members. The other actions may be appropriate, but this will depend on what is learned when assessing family preferences. c) A 22-year-old with multiple fractures of

These four patients arrive in the

the face and jaw

emergency department after a motor

a) A 72-year-old with palpitations and

vehicle crash. In which order should

chest pain

they be assessed?

b) A 45-year-old complaining of 6/10 abdominal pain d) A 30-year-old with a misaligned right leg with intact pulses rationale: The highest priority is to assess the 22year-old patient for airway obstruction, which is the most life-threatening injury. The 72-year-old patient may have chest pain from cardiac ischemia and should be assessed and have diagnostic testing for this pain. The 45-year-old patient may have abdominal trauma or bleeding and should be seen next to assess circulatory status. The 30-year-old appears to have a possible fracture of the right leg and should be seen soon, but this patient has

the least life-threatening injury.

View more...

Comments

Copyright ©2017 KUPDF Inc.
SUPPORT KUPDF